Difference between revisions of "1959 IMO Problems/Problem 1"

m (correction in problem)
Line 4: Line 4:
  
  
== Solution ==
+
== Solutions ==
 +
 
 +
=== First Solution ===
  
 
We observe that
 
We observe that
  
 
<center>
 
<center>
<math>\displaystyle 3(14n+3) = 2(21n+4) + 1,</math>
+
<math>\displaystyle 3(14n+3) = 2(21n+4) + 1.</math>
 
</center>
 
</center>
  
and we're done.
 
  
 +
Since a multiple of <math>\displaystyle 14n+3</math> differs from a multiple of <math>\displaystyle 21n+4</math> by 1, we cannot have any postive integer greater than 1 simultaneously divide <math>\displaystyle 14n+3</math> and <math>\displaystyle 21n+4</math>.  Hence the [[greatest common divisor]] of the fraction's numerator and denominator is 1, so the fraction is irreducible.  Q.E.D.
 +
 +
=== Second Solution ===
 +
 +
Denoting the greatest common divisor of <math>\displaystyle a, b </math> as <math>\displaystyle (a,b) </math>, we use the [[Euclidean algorithm]] as follows:
 +
 +
<center>
 +
<math>\displaystyle ( 21n+4, 14n+3 ) = ( 7n+1, 14n+3 ) = ( 7n+1, 1 ) = 1 </math>
 +
</center>
 +
 +
As in the first solution, it follows that <math>\frac{21n+4}{14n+3}</math> is irreducible.  Q.E.D.
  
 
== Resources ==
 
== Resources ==

Revision as of 20:16, 25 July 2006

Problem

Prove that the fraction $\frac{21n+4}{14n+3}$ is irreducible for every natural number $\displaystyle n$.


Solutions

First Solution

We observe that

$\displaystyle 3(14n+3) = 2(21n+4) + 1.$


Since a multiple of $\displaystyle 14n+3$ differs from a multiple of $\displaystyle 21n+4$ by 1, we cannot have any postive integer greater than 1 simultaneously divide $\displaystyle 14n+3$ and $\displaystyle 21n+4$. Hence the greatest common divisor of the fraction's numerator and denominator is 1, so the fraction is irreducible. Q.E.D.

Second Solution

Denoting the greatest common divisor of $\displaystyle a, b$ as $\displaystyle (a,b)$, we use the Euclidean algorithm as follows:

$\displaystyle ( 21n+4, 14n+3 ) = ( 7n+1, 14n+3 ) = ( 7n+1, 1 ) = 1$

As in the first solution, it follows that $\frac{21n+4}{14n+3}$ is irreducible. Q.E.D.

Resources